Difference between revisions of "2007 AMC 8 Problems/Problem 18"

m (Answer)
m
 
(14 intermediate revisions by 14 users not shown)
Line 1: Line 1:
Answer is b
+
==Problem==
  
The question asks the person what the sum of the thousands digit is and the units digit from the second number,
+
The product of the two <math>99</math>-digit numbers
  
therefore using 5+0=5, you should get the answer 5
+
<math>303,030,303,...,030,303</math> and <math>505,050,505,...,050,505</math>
  
 +
has thousands digit <math>A</math> and units digit <math>B</math>. What is the sum of <math>A</math> and <math>B</math>?
  
 +
<math>\mathrm{(A)}\ 3 \qquad \mathrm{(B)}\ 5 \qquad \mathrm{(C)}\ 6 \qquad \mathrm{(D)}\ 8 \qquad \mathrm{(E)}\ 10</math>
  
Answer written by a third party person
+
 
Divyesh doddapaneni
+
==Solution==
--[[User:Divyesh|Divyesh]] 01:34, 1 November 2010 (UTC)
+
We can first make a small example to find out <math>A</math> and <math>B</math>. So,
 +
 
 +
<math>303\times505=153015 </math>
 +
 
 +
The ones digit plus thousands digit is <math>5+3=8</math>.
 +
 
 +
Note that the ones and thousands digits are, added together, <math>8</math>. (and so on...) So the answer is <math>\boxed{\textbf{(D)}\ 8}</math>
 +
This is a direct multiplication way.
 +
 
 +
==Video Solution by OmegaLearn==
 +
https://youtu.be/7an5wU9Q5hk?t=2085
 +
 
 +
~ pi_is_3.14
 +
 
 +
==Video Solution by WhyMath==
 +
https://youtu.be/_goaFuScO6M
 +
 
 +
~savannahsolver
 +
 
 +
==See Also==
 +
{{AMC8 box|year=2007|num-b=17|num-a=19}}
 +
{{MAA Notice}}

Latest revision as of 05:43, 31 December 2022

Problem

The product of the two $99$-digit numbers

$303,030,303,...,030,303$ and $505,050,505,...,050,505$

has thousands digit $A$ and units digit $B$. What is the sum of $A$ and $B$?

$\mathrm{(A)}\ 3 \qquad \mathrm{(B)}\ 5 \qquad \mathrm{(C)}\ 6 \qquad \mathrm{(D)}\ 8 \qquad \mathrm{(E)}\ 10$


Solution

We can first make a small example to find out $A$ and $B$. So,

$303\times505=153015$

The ones digit plus thousands digit is $5+3=8$.

Note that the ones and thousands digits are, added together, $8$. (and so on...) So the answer is $\boxed{\textbf{(D)}\ 8}$ This is a direct multiplication way.

Video Solution by OmegaLearn

https://youtu.be/7an5wU9Q5hk?t=2085

~ pi_is_3.14

Video Solution by WhyMath

https://youtu.be/_goaFuScO6M

~savannahsolver

See Also

2007 AMC 8 (ProblemsAnswer KeyResources)
Preceded by
Problem 17
Followed by
Problem 19
1 2 3 4 5 6 7 8 9 10 11 12 13 14 15 16 17 18 19 20 21 22 23 24 25
All AJHSME/AMC 8 Problems and Solutions

The problems on this page are copyrighted by the Mathematical Association of America's American Mathematics Competitions. AMC logo.png